$a^2−ab+b^2=c^2$. Show prime factors of $c$ are of the form $6k + 1$

The name of the pictureThe name of the pictureThe name of the pictureClash Royale CLAN TAG#URR8PPP











up vote
2
down vote

favorite












I encountered this question in Number Theory by Naoki Sato.



Let a, b, and c be positive integers that are pairwise relatively prime,
and that satisfy $a^2−ab+b^2=c^2$. Show that every prime factor of c is
of the form $6k + 1$.



Attempt:



Since primes are either of the form $6k+1$ or $6k-1$, I tried to show $p equiv 1 pmod6$ by showing $p equiv 1 pmod2$ and $p equiv 1 pmod3$ but I am unable to show $p equiv 1 pmod3$.



Another direction I tried was to assume there exist a prime factor $p$ of $c$ with $p=6k-1$.
By brute force I know that $a^2−ab+b^2 equiv 0, 1, 3$, or $4 pmod6$ but I can't see how to get a contradiction from here.



I am not sure how to use the condition that all 3 numbers are pairwise coprime.



Any help is much appreciated!







share|cite|improve this question















  • 1




    math.stackexchange.com/questions/1351994/…
    – individ
    Jul 18 at 17:00






  • 2




    Have you ever seen the "Gaussian" proofs that if $a^2+b^2=c$ and $a, b, c$ are relatively prime, then all prime factors of $c$ are of the form $4k+1$? This operates on very similar principles using a different ring of integers.
    – Steven Stadnicki
    Jul 18 at 17:00










  • @individ thanks for the link! ill check it out
    – eatfood
    Jul 19 at 3:19










  • @StevenStadnicki no I haven't, I have only just started on learning number theory, and some of the proofs seem very difficult to come up with
    – eatfood
    Jul 19 at 3:20














up vote
2
down vote

favorite












I encountered this question in Number Theory by Naoki Sato.



Let a, b, and c be positive integers that are pairwise relatively prime,
and that satisfy $a^2−ab+b^2=c^2$. Show that every prime factor of c is
of the form $6k + 1$.



Attempt:



Since primes are either of the form $6k+1$ or $6k-1$, I tried to show $p equiv 1 pmod6$ by showing $p equiv 1 pmod2$ and $p equiv 1 pmod3$ but I am unable to show $p equiv 1 pmod3$.



Another direction I tried was to assume there exist a prime factor $p$ of $c$ with $p=6k-1$.
By brute force I know that $a^2−ab+b^2 equiv 0, 1, 3$, or $4 pmod6$ but I can't see how to get a contradiction from here.



I am not sure how to use the condition that all 3 numbers are pairwise coprime.



Any help is much appreciated!







share|cite|improve this question















  • 1




    math.stackexchange.com/questions/1351994/…
    – individ
    Jul 18 at 17:00






  • 2




    Have you ever seen the "Gaussian" proofs that if $a^2+b^2=c$ and $a, b, c$ are relatively prime, then all prime factors of $c$ are of the form $4k+1$? This operates on very similar principles using a different ring of integers.
    – Steven Stadnicki
    Jul 18 at 17:00










  • @individ thanks for the link! ill check it out
    – eatfood
    Jul 19 at 3:19










  • @StevenStadnicki no I haven't, I have only just started on learning number theory, and some of the proofs seem very difficult to come up with
    – eatfood
    Jul 19 at 3:20












up vote
2
down vote

favorite









up vote
2
down vote

favorite











I encountered this question in Number Theory by Naoki Sato.



Let a, b, and c be positive integers that are pairwise relatively prime,
and that satisfy $a^2−ab+b^2=c^2$. Show that every prime factor of c is
of the form $6k + 1$.



Attempt:



Since primes are either of the form $6k+1$ or $6k-1$, I tried to show $p equiv 1 pmod6$ by showing $p equiv 1 pmod2$ and $p equiv 1 pmod3$ but I am unable to show $p equiv 1 pmod3$.



Another direction I tried was to assume there exist a prime factor $p$ of $c$ with $p=6k-1$.
By brute force I know that $a^2−ab+b^2 equiv 0, 1, 3$, or $4 pmod6$ but I can't see how to get a contradiction from here.



I am not sure how to use the condition that all 3 numbers are pairwise coprime.



Any help is much appreciated!







share|cite|improve this question











I encountered this question in Number Theory by Naoki Sato.



Let a, b, and c be positive integers that are pairwise relatively prime,
and that satisfy $a^2−ab+b^2=c^2$. Show that every prime factor of c is
of the form $6k + 1$.



Attempt:



Since primes are either of the form $6k+1$ or $6k-1$, I tried to show $p equiv 1 pmod6$ by showing $p equiv 1 pmod2$ and $p equiv 1 pmod3$ but I am unable to show $p equiv 1 pmod3$.



Another direction I tried was to assume there exist a prime factor $p$ of $c$ with $p=6k-1$.
By brute force I know that $a^2−ab+b^2 equiv 0, 1, 3$, or $4 pmod6$ but I can't see how to get a contradiction from here.



I am not sure how to use the condition that all 3 numbers are pairwise coprime.



Any help is much appreciated!









share|cite|improve this question










share|cite|improve this question




share|cite|improve this question









asked Jul 18 at 16:53









eatfood

452




452







  • 1




    math.stackexchange.com/questions/1351994/…
    – individ
    Jul 18 at 17:00






  • 2




    Have you ever seen the "Gaussian" proofs that if $a^2+b^2=c$ and $a, b, c$ are relatively prime, then all prime factors of $c$ are of the form $4k+1$? This operates on very similar principles using a different ring of integers.
    – Steven Stadnicki
    Jul 18 at 17:00










  • @individ thanks for the link! ill check it out
    – eatfood
    Jul 19 at 3:19










  • @StevenStadnicki no I haven't, I have only just started on learning number theory, and some of the proofs seem very difficult to come up with
    – eatfood
    Jul 19 at 3:20












  • 1




    math.stackexchange.com/questions/1351994/…
    – individ
    Jul 18 at 17:00






  • 2




    Have you ever seen the "Gaussian" proofs that if $a^2+b^2=c$ and $a, b, c$ are relatively prime, then all prime factors of $c$ are of the form $4k+1$? This operates on very similar principles using a different ring of integers.
    – Steven Stadnicki
    Jul 18 at 17:00










  • @individ thanks for the link! ill check it out
    – eatfood
    Jul 19 at 3:19










  • @StevenStadnicki no I haven't, I have only just started on learning number theory, and some of the proofs seem very difficult to come up with
    – eatfood
    Jul 19 at 3:20







1




1




math.stackexchange.com/questions/1351994/…
– individ
Jul 18 at 17:00




math.stackexchange.com/questions/1351994/…
– individ
Jul 18 at 17:00




2




2




Have you ever seen the "Gaussian" proofs that if $a^2+b^2=c$ and $a, b, c$ are relatively prime, then all prime factors of $c$ are of the form $4k+1$? This operates on very similar principles using a different ring of integers.
– Steven Stadnicki
Jul 18 at 17:00




Have you ever seen the "Gaussian" proofs that if $a^2+b^2=c$ and $a, b, c$ are relatively prime, then all prime factors of $c$ are of the form $4k+1$? This operates on very similar principles using a different ring of integers.
– Steven Stadnicki
Jul 18 at 17:00












@individ thanks for the link! ill check it out
– eatfood
Jul 19 at 3:19




@individ thanks for the link! ill check it out
– eatfood
Jul 19 at 3:19












@StevenStadnicki no I haven't, I have only just started on learning number theory, and some of the proofs seem very difficult to come up with
– eatfood
Jul 19 at 3:20




@StevenStadnicki no I haven't, I have only just started on learning number theory, and some of the proofs seem very difficult to come up with
– eatfood
Jul 19 at 3:20










3 Answers
3






active

oldest

votes

















up vote
1
down vote



accepted










Here is a proof based on quadratic reciprocity.



Suppose $p=3k-1|a^2-ab+b^2$ with $a,b$ relatively prime. Neither $a$ nor $b$ can have zero residue $bmod p$, else they both would which the hypothesis denies. Thus there exists a nonzero residue $requiv ab^-1bmod p$ such that $r^2-r+1equiv 0$. Multiplying by $4$ and completing the square gives $(2r-1)^2equiv -3$. Therefore the Legendre symbol $(-3|p)=+1$ if $p$ is as assumed above.



But $(p|3)=(3k-1|3)=-1$ and $-3equiv +1bmod 4$, so QR forces the contradictory conclusion $(-3|p)=-1$.



The case $p=3$ remains and this is a bit tricky. There are multiples of $3$ having the form $a^2-ab+b^2$ where $a$ and $b$ are relatively prime. We just need $aequiv -bbmod 3$ because $a^2-ab+b^2equiv (a+b)^2bmod 3$. But, if $aequiv -bbmod 3$ with the common residue being $1$ or $2$, we find that $c^2equiv 3bmod 9$ which doesn't work.






share|cite|improve this answer



















  • 1




    Hi Oscar, this is a really neat way to solve it. I got to $r^2 - r + 1 equiv 0 pmod p$ but I did not know how to proceed further. Completing the square to introduce the Legendre symbol with 3 as the other prime is really neat. Thanks alot!
    – eatfood
    Jul 19 at 3:15











  • So more generally, for $p ne 2$ prime (and $a notequiv 0 pmodp$), $ar^2 + br + c equiv 0 pmodp$ has solutions if and only if $(b^2-4ac | p) = 1$. (Slightly different proof to the completing squares one: if the two roots are $r_1,r_2$ then $b^2-4ac = [a (r_1 - r_2)]^2$.)
    – Daniel Schepler
    Jul 19 at 15:32










  • One degenerate solution if $(b^2-4ac|p)=0$, e.g. $x^2-x-1equiv 0bmod 5$ where $b^2-4ac=5equiv 0$ has a solution $xequiv 3$. The existence of such a solution in the case of this problem is why we had to treat $p=3$ separately.
    – Oscar Lanzi
    Jul 19 at 21:20


















up vote
3
down vote













Suppose $p mid c$, so that $a^2 - ab + b^2 equiv 0 pmodp$. That implies that $a^3 + b^3 equiv 0 pmodp$. By the assumption that $a$ and $b$ are each relatively prime to $c$, we have that $p nmid a, b$, so $-a b^-1$ has cube equal to 1 in $mathbbZ / p mathbbZ$. Also, if it were the case that $a equiv -b pmodp$, then we would have $3 a^2 equiv a^2 - ab + b^2 equiv 0 pmodp$, giving a contradiction (as long as $pne 3$; but in this case, for example if $a = 3m+1$ and $b=3n-1$, then $a^2 - ab + b^2 = 9m^2 - 9mn + 9n^2 -9m + 9n + 3 equiv 3 pmod9$ contradicting the fact that $3 mid c^2$ implies $9 mid c^2$). Therefore, $-a b^-1$ has order exactly three in the group of units of $mathbbZ / p mathbbZ$, implying that 3 divides $|(mathbbZ / p mathbbZ)^*| = p-1$.






share|cite|improve this answer



















  • 1




    Hi Daniel, that is a really natural way to solve it. I did not even think of $a^3 + b^3 equiv 0 pmod p$. Although i really like this solution, I accepted the Oscar's answer because the problem is located in the QR section of the handout so I'm assuming his solution is the one intended by the author. Thanks very much still!
    – eatfood
    Jul 19 at 3:12

















up vote
0
down vote













Here is an algebraic theoretical proof parallel to the "gaussian" proof of Fermat's theorem on sums of squares. You may find it too complicated, but actually the general trend in math is to try to embed particular questions into larger and larger general theories in order to solve in one stroke whole families of problems which are of the same nature (*)



First replace the Gauss ring $mathbf Z[i]$ by the Eisenstein ring $E=mathbf Z[j]$, which is the ring of integers of the quadratic field $mathbf Q(j)$, where $j$ is a primitive 3rd root of unity. It is classically known that $E$ is a principal ideal ring, so the theory of division in $E$ for ideals and for elements is the same up to the units of $E$, which are plainly the powers of $pm j$. The prime elements (ideals) of $E$ are deduced from those of $mathbf Z$ in the following manner : a prime $p$ of $mathbf Z$ does not necessarily remain prime in $E$, but it can decompose only in 3 possible ways : (1) $p$ is inert, i.e. remains prime, iff $pequiv 2$ mod $3$ ; (2) $p$ splits, i.e. $p=pibarpi$ (where $pi$ is a prime of $E$) iff $pequiv 1$ mod $3$ ; (3) $p$ ramifies in $E$, i.e. $p=pi^2$ iff $p=3$ (see any textbook in ANT).



Now the equation $a^2-ab+b^2=c^2$ in $mathbf Z$ reads $(a+bj)(a+bj^2)=c^2$ in $E$, where the UFD property implies that the prime divisors $pi$ of $c$ are exactly such that $pi$ divides $a+bj$, say, and the conjugate $bar pi$ divides $a+bj^2$. By uniqueness again, $pibar pi$ will divide $c$, i.e. the rational primes $p$ which divide $c$ must split.



(*) An archetypical example lies in the attempts to solve FLT by the same approach as above. Actually Kummer showed FLT along these lines - but of course with more involved arguments - in the so called regular case, i.e. when the ring $mathbf Z[zeta_p]$ (where $zeta_p$ is a primitive $p$-th root of unity) is principal.






share|cite|improve this answer





















    Your Answer




    StackExchange.ifUsing("editor", function ()
    return StackExchange.using("mathjaxEditing", function ()
    StackExchange.MarkdownEditor.creationCallbacks.add(function (editor, postfix)
    StackExchange.mathjaxEditing.prepareWmdForMathJax(editor, postfix, [["$", "$"], ["\\(","\\)"]]);
    );
    );
    , "mathjax-editing");

    StackExchange.ready(function()
    var channelOptions =
    tags: "".split(" "),
    id: "69"
    ;
    initTagRenderer("".split(" "), "".split(" "), channelOptions);

    StackExchange.using("externalEditor", function()
    // Have to fire editor after snippets, if snippets enabled
    if (StackExchange.settings.snippets.snippetsEnabled)
    StackExchange.using("snippets", function()
    createEditor();
    );

    else
    createEditor();

    );

    function createEditor()
    StackExchange.prepareEditor(
    heartbeatType: 'answer',
    convertImagesToLinks: true,
    noModals: false,
    showLowRepImageUploadWarning: true,
    reputationToPostImages: 10,
    bindNavPrevention: true,
    postfix: "",
    noCode: true, onDemand: true,
    discardSelector: ".discard-answer"
    ,immediatelyShowMarkdownHelp:true
    );



    );








     

    draft saved


    draft discarded


















    StackExchange.ready(
    function ()
    StackExchange.openid.initPostLogin('.new-post-login', 'https%3a%2f%2fmath.stackexchange.com%2fquestions%2f2855769%2fa2%25e2%2588%2592abb2-c2-show-prime-factors-of-c-are-of-the-form-6k-1%23new-answer', 'question_page');

    );

    Post as a guest






























    3 Answers
    3






    active

    oldest

    votes








    3 Answers
    3






    active

    oldest

    votes









    active

    oldest

    votes






    active

    oldest

    votes








    up vote
    1
    down vote



    accepted










    Here is a proof based on quadratic reciprocity.



    Suppose $p=3k-1|a^2-ab+b^2$ with $a,b$ relatively prime. Neither $a$ nor $b$ can have zero residue $bmod p$, else they both would which the hypothesis denies. Thus there exists a nonzero residue $requiv ab^-1bmod p$ such that $r^2-r+1equiv 0$. Multiplying by $4$ and completing the square gives $(2r-1)^2equiv -3$. Therefore the Legendre symbol $(-3|p)=+1$ if $p$ is as assumed above.



    But $(p|3)=(3k-1|3)=-1$ and $-3equiv +1bmod 4$, so QR forces the contradictory conclusion $(-3|p)=-1$.



    The case $p=3$ remains and this is a bit tricky. There are multiples of $3$ having the form $a^2-ab+b^2$ where $a$ and $b$ are relatively prime. We just need $aequiv -bbmod 3$ because $a^2-ab+b^2equiv (a+b)^2bmod 3$. But, if $aequiv -bbmod 3$ with the common residue being $1$ or $2$, we find that $c^2equiv 3bmod 9$ which doesn't work.






    share|cite|improve this answer



















    • 1




      Hi Oscar, this is a really neat way to solve it. I got to $r^2 - r + 1 equiv 0 pmod p$ but I did not know how to proceed further. Completing the square to introduce the Legendre symbol with 3 as the other prime is really neat. Thanks alot!
      – eatfood
      Jul 19 at 3:15











    • So more generally, for $p ne 2$ prime (and $a notequiv 0 pmodp$), $ar^2 + br + c equiv 0 pmodp$ has solutions if and only if $(b^2-4ac | p) = 1$. (Slightly different proof to the completing squares one: if the two roots are $r_1,r_2$ then $b^2-4ac = [a (r_1 - r_2)]^2$.)
      – Daniel Schepler
      Jul 19 at 15:32










    • One degenerate solution if $(b^2-4ac|p)=0$, e.g. $x^2-x-1equiv 0bmod 5$ where $b^2-4ac=5equiv 0$ has a solution $xequiv 3$. The existence of such a solution in the case of this problem is why we had to treat $p=3$ separately.
      – Oscar Lanzi
      Jul 19 at 21:20















    up vote
    1
    down vote



    accepted










    Here is a proof based on quadratic reciprocity.



    Suppose $p=3k-1|a^2-ab+b^2$ with $a,b$ relatively prime. Neither $a$ nor $b$ can have zero residue $bmod p$, else they both would which the hypothesis denies. Thus there exists a nonzero residue $requiv ab^-1bmod p$ such that $r^2-r+1equiv 0$. Multiplying by $4$ and completing the square gives $(2r-1)^2equiv -3$. Therefore the Legendre symbol $(-3|p)=+1$ if $p$ is as assumed above.



    But $(p|3)=(3k-1|3)=-1$ and $-3equiv +1bmod 4$, so QR forces the contradictory conclusion $(-3|p)=-1$.



    The case $p=3$ remains and this is a bit tricky. There are multiples of $3$ having the form $a^2-ab+b^2$ where $a$ and $b$ are relatively prime. We just need $aequiv -bbmod 3$ because $a^2-ab+b^2equiv (a+b)^2bmod 3$. But, if $aequiv -bbmod 3$ with the common residue being $1$ or $2$, we find that $c^2equiv 3bmod 9$ which doesn't work.






    share|cite|improve this answer



















    • 1




      Hi Oscar, this is a really neat way to solve it. I got to $r^2 - r + 1 equiv 0 pmod p$ but I did not know how to proceed further. Completing the square to introduce the Legendre symbol with 3 as the other prime is really neat. Thanks alot!
      – eatfood
      Jul 19 at 3:15











    • So more generally, for $p ne 2$ prime (and $a notequiv 0 pmodp$), $ar^2 + br + c equiv 0 pmodp$ has solutions if and only if $(b^2-4ac | p) = 1$. (Slightly different proof to the completing squares one: if the two roots are $r_1,r_2$ then $b^2-4ac = [a (r_1 - r_2)]^2$.)
      – Daniel Schepler
      Jul 19 at 15:32










    • One degenerate solution if $(b^2-4ac|p)=0$, e.g. $x^2-x-1equiv 0bmod 5$ where $b^2-4ac=5equiv 0$ has a solution $xequiv 3$. The existence of such a solution in the case of this problem is why we had to treat $p=3$ separately.
      – Oscar Lanzi
      Jul 19 at 21:20













    up vote
    1
    down vote



    accepted







    up vote
    1
    down vote



    accepted






    Here is a proof based on quadratic reciprocity.



    Suppose $p=3k-1|a^2-ab+b^2$ with $a,b$ relatively prime. Neither $a$ nor $b$ can have zero residue $bmod p$, else they both would which the hypothesis denies. Thus there exists a nonzero residue $requiv ab^-1bmod p$ such that $r^2-r+1equiv 0$. Multiplying by $4$ and completing the square gives $(2r-1)^2equiv -3$. Therefore the Legendre symbol $(-3|p)=+1$ if $p$ is as assumed above.



    But $(p|3)=(3k-1|3)=-1$ and $-3equiv +1bmod 4$, so QR forces the contradictory conclusion $(-3|p)=-1$.



    The case $p=3$ remains and this is a bit tricky. There are multiples of $3$ having the form $a^2-ab+b^2$ where $a$ and $b$ are relatively prime. We just need $aequiv -bbmod 3$ because $a^2-ab+b^2equiv (a+b)^2bmod 3$. But, if $aequiv -bbmod 3$ with the common residue being $1$ or $2$, we find that $c^2equiv 3bmod 9$ which doesn't work.






    share|cite|improve this answer















    Here is a proof based on quadratic reciprocity.



    Suppose $p=3k-1|a^2-ab+b^2$ with $a,b$ relatively prime. Neither $a$ nor $b$ can have zero residue $bmod p$, else they both would which the hypothesis denies. Thus there exists a nonzero residue $requiv ab^-1bmod p$ such that $r^2-r+1equiv 0$. Multiplying by $4$ and completing the square gives $(2r-1)^2equiv -3$. Therefore the Legendre symbol $(-3|p)=+1$ if $p$ is as assumed above.



    But $(p|3)=(3k-1|3)=-1$ and $-3equiv +1bmod 4$, so QR forces the contradictory conclusion $(-3|p)=-1$.



    The case $p=3$ remains and this is a bit tricky. There are multiples of $3$ having the form $a^2-ab+b^2$ where $a$ and $b$ are relatively prime. We just need $aequiv -bbmod 3$ because $a^2-ab+b^2equiv (a+b)^2bmod 3$. But, if $aequiv -bbmod 3$ with the common residue being $1$ or $2$, we find that $c^2equiv 3bmod 9$ which doesn't work.







    share|cite|improve this answer















    share|cite|improve this answer



    share|cite|improve this answer








    edited Jul 23 at 1:43


























    answered Jul 18 at 17:25









    Oscar Lanzi

    9,98911632




    9,98911632







    • 1




      Hi Oscar, this is a really neat way to solve it. I got to $r^2 - r + 1 equiv 0 pmod p$ but I did not know how to proceed further. Completing the square to introduce the Legendre symbol with 3 as the other prime is really neat. Thanks alot!
      – eatfood
      Jul 19 at 3:15











    • So more generally, for $p ne 2$ prime (and $a notequiv 0 pmodp$), $ar^2 + br + c equiv 0 pmodp$ has solutions if and only if $(b^2-4ac | p) = 1$. (Slightly different proof to the completing squares one: if the two roots are $r_1,r_2$ then $b^2-4ac = [a (r_1 - r_2)]^2$.)
      – Daniel Schepler
      Jul 19 at 15:32










    • One degenerate solution if $(b^2-4ac|p)=0$, e.g. $x^2-x-1equiv 0bmod 5$ where $b^2-4ac=5equiv 0$ has a solution $xequiv 3$. The existence of such a solution in the case of this problem is why we had to treat $p=3$ separately.
      – Oscar Lanzi
      Jul 19 at 21:20













    • 1




      Hi Oscar, this is a really neat way to solve it. I got to $r^2 - r + 1 equiv 0 pmod p$ but I did not know how to proceed further. Completing the square to introduce the Legendre symbol with 3 as the other prime is really neat. Thanks alot!
      – eatfood
      Jul 19 at 3:15











    • So more generally, for $p ne 2$ prime (and $a notequiv 0 pmodp$), $ar^2 + br + c equiv 0 pmodp$ has solutions if and only if $(b^2-4ac | p) = 1$. (Slightly different proof to the completing squares one: if the two roots are $r_1,r_2$ then $b^2-4ac = [a (r_1 - r_2)]^2$.)
      – Daniel Schepler
      Jul 19 at 15:32










    • One degenerate solution if $(b^2-4ac|p)=0$, e.g. $x^2-x-1equiv 0bmod 5$ where $b^2-4ac=5equiv 0$ has a solution $xequiv 3$. The existence of such a solution in the case of this problem is why we had to treat $p=3$ separately.
      – Oscar Lanzi
      Jul 19 at 21:20








    1




    1




    Hi Oscar, this is a really neat way to solve it. I got to $r^2 - r + 1 equiv 0 pmod p$ but I did not know how to proceed further. Completing the square to introduce the Legendre symbol with 3 as the other prime is really neat. Thanks alot!
    – eatfood
    Jul 19 at 3:15





    Hi Oscar, this is a really neat way to solve it. I got to $r^2 - r + 1 equiv 0 pmod p$ but I did not know how to proceed further. Completing the square to introduce the Legendre symbol with 3 as the other prime is really neat. Thanks alot!
    – eatfood
    Jul 19 at 3:15













    So more generally, for $p ne 2$ prime (and $a notequiv 0 pmodp$), $ar^2 + br + c equiv 0 pmodp$ has solutions if and only if $(b^2-4ac | p) = 1$. (Slightly different proof to the completing squares one: if the two roots are $r_1,r_2$ then $b^2-4ac = [a (r_1 - r_2)]^2$.)
    – Daniel Schepler
    Jul 19 at 15:32




    So more generally, for $p ne 2$ prime (and $a notequiv 0 pmodp$), $ar^2 + br + c equiv 0 pmodp$ has solutions if and only if $(b^2-4ac | p) = 1$. (Slightly different proof to the completing squares one: if the two roots are $r_1,r_2$ then $b^2-4ac = [a (r_1 - r_2)]^2$.)
    – Daniel Schepler
    Jul 19 at 15:32












    One degenerate solution if $(b^2-4ac|p)=0$, e.g. $x^2-x-1equiv 0bmod 5$ where $b^2-4ac=5equiv 0$ has a solution $xequiv 3$. The existence of such a solution in the case of this problem is why we had to treat $p=3$ separately.
    – Oscar Lanzi
    Jul 19 at 21:20





    One degenerate solution if $(b^2-4ac|p)=0$, e.g. $x^2-x-1equiv 0bmod 5$ where $b^2-4ac=5equiv 0$ has a solution $xequiv 3$. The existence of such a solution in the case of this problem is why we had to treat $p=3$ separately.
    – Oscar Lanzi
    Jul 19 at 21:20











    up vote
    3
    down vote













    Suppose $p mid c$, so that $a^2 - ab + b^2 equiv 0 pmodp$. That implies that $a^3 + b^3 equiv 0 pmodp$. By the assumption that $a$ and $b$ are each relatively prime to $c$, we have that $p nmid a, b$, so $-a b^-1$ has cube equal to 1 in $mathbbZ / p mathbbZ$. Also, if it were the case that $a equiv -b pmodp$, then we would have $3 a^2 equiv a^2 - ab + b^2 equiv 0 pmodp$, giving a contradiction (as long as $pne 3$; but in this case, for example if $a = 3m+1$ and $b=3n-1$, then $a^2 - ab + b^2 = 9m^2 - 9mn + 9n^2 -9m + 9n + 3 equiv 3 pmod9$ contradicting the fact that $3 mid c^2$ implies $9 mid c^2$). Therefore, $-a b^-1$ has order exactly three in the group of units of $mathbbZ / p mathbbZ$, implying that 3 divides $|(mathbbZ / p mathbbZ)^*| = p-1$.






    share|cite|improve this answer



















    • 1




      Hi Daniel, that is a really natural way to solve it. I did not even think of $a^3 + b^3 equiv 0 pmod p$. Although i really like this solution, I accepted the Oscar's answer because the problem is located in the QR section of the handout so I'm assuming his solution is the one intended by the author. Thanks very much still!
      – eatfood
      Jul 19 at 3:12














    up vote
    3
    down vote













    Suppose $p mid c$, so that $a^2 - ab + b^2 equiv 0 pmodp$. That implies that $a^3 + b^3 equiv 0 pmodp$. By the assumption that $a$ and $b$ are each relatively prime to $c$, we have that $p nmid a, b$, so $-a b^-1$ has cube equal to 1 in $mathbbZ / p mathbbZ$. Also, if it were the case that $a equiv -b pmodp$, then we would have $3 a^2 equiv a^2 - ab + b^2 equiv 0 pmodp$, giving a contradiction (as long as $pne 3$; but in this case, for example if $a = 3m+1$ and $b=3n-1$, then $a^2 - ab + b^2 = 9m^2 - 9mn + 9n^2 -9m + 9n + 3 equiv 3 pmod9$ contradicting the fact that $3 mid c^2$ implies $9 mid c^2$). Therefore, $-a b^-1$ has order exactly three in the group of units of $mathbbZ / p mathbbZ$, implying that 3 divides $|(mathbbZ / p mathbbZ)^*| = p-1$.






    share|cite|improve this answer



















    • 1




      Hi Daniel, that is a really natural way to solve it. I did not even think of $a^3 + b^3 equiv 0 pmod p$. Although i really like this solution, I accepted the Oscar's answer because the problem is located in the QR section of the handout so I'm assuming his solution is the one intended by the author. Thanks very much still!
      – eatfood
      Jul 19 at 3:12












    up vote
    3
    down vote










    up vote
    3
    down vote









    Suppose $p mid c$, so that $a^2 - ab + b^2 equiv 0 pmodp$. That implies that $a^3 + b^3 equiv 0 pmodp$. By the assumption that $a$ and $b$ are each relatively prime to $c$, we have that $p nmid a, b$, so $-a b^-1$ has cube equal to 1 in $mathbbZ / p mathbbZ$. Also, if it were the case that $a equiv -b pmodp$, then we would have $3 a^2 equiv a^2 - ab + b^2 equiv 0 pmodp$, giving a contradiction (as long as $pne 3$; but in this case, for example if $a = 3m+1$ and $b=3n-1$, then $a^2 - ab + b^2 = 9m^2 - 9mn + 9n^2 -9m + 9n + 3 equiv 3 pmod9$ contradicting the fact that $3 mid c^2$ implies $9 mid c^2$). Therefore, $-a b^-1$ has order exactly three in the group of units of $mathbbZ / p mathbbZ$, implying that 3 divides $|(mathbbZ / p mathbbZ)^*| = p-1$.






    share|cite|improve this answer















    Suppose $p mid c$, so that $a^2 - ab + b^2 equiv 0 pmodp$. That implies that $a^3 + b^3 equiv 0 pmodp$. By the assumption that $a$ and $b$ are each relatively prime to $c$, we have that $p nmid a, b$, so $-a b^-1$ has cube equal to 1 in $mathbbZ / p mathbbZ$. Also, if it were the case that $a equiv -b pmodp$, then we would have $3 a^2 equiv a^2 - ab + b^2 equiv 0 pmodp$, giving a contradiction (as long as $pne 3$; but in this case, for example if $a = 3m+1$ and $b=3n-1$, then $a^2 - ab + b^2 = 9m^2 - 9mn + 9n^2 -9m + 9n + 3 equiv 3 pmod9$ contradicting the fact that $3 mid c^2$ implies $9 mid c^2$). Therefore, $-a b^-1$ has order exactly three in the group of units of $mathbbZ / p mathbbZ$, implying that 3 divides $|(mathbbZ / p mathbbZ)^*| = p-1$.







    share|cite|improve this answer















    share|cite|improve this answer



    share|cite|improve this answer








    edited Jul 18 at 17:50


























    answered Jul 18 at 17:15









    Daniel Schepler

    6,9131514




    6,9131514







    • 1




      Hi Daniel, that is a really natural way to solve it. I did not even think of $a^3 + b^3 equiv 0 pmod p$. Although i really like this solution, I accepted the Oscar's answer because the problem is located in the QR section of the handout so I'm assuming his solution is the one intended by the author. Thanks very much still!
      – eatfood
      Jul 19 at 3:12












    • 1




      Hi Daniel, that is a really natural way to solve it. I did not even think of $a^3 + b^3 equiv 0 pmod p$. Although i really like this solution, I accepted the Oscar's answer because the problem is located in the QR section of the handout so I'm assuming his solution is the one intended by the author. Thanks very much still!
      – eatfood
      Jul 19 at 3:12







    1




    1




    Hi Daniel, that is a really natural way to solve it. I did not even think of $a^3 + b^3 equiv 0 pmod p$. Although i really like this solution, I accepted the Oscar's answer because the problem is located in the QR section of the handout so I'm assuming his solution is the one intended by the author. Thanks very much still!
    – eatfood
    Jul 19 at 3:12




    Hi Daniel, that is a really natural way to solve it. I did not even think of $a^3 + b^3 equiv 0 pmod p$. Although i really like this solution, I accepted the Oscar's answer because the problem is located in the QR section of the handout so I'm assuming his solution is the one intended by the author. Thanks very much still!
    – eatfood
    Jul 19 at 3:12










    up vote
    0
    down vote













    Here is an algebraic theoretical proof parallel to the "gaussian" proof of Fermat's theorem on sums of squares. You may find it too complicated, but actually the general trend in math is to try to embed particular questions into larger and larger general theories in order to solve in one stroke whole families of problems which are of the same nature (*)



    First replace the Gauss ring $mathbf Z[i]$ by the Eisenstein ring $E=mathbf Z[j]$, which is the ring of integers of the quadratic field $mathbf Q(j)$, where $j$ is a primitive 3rd root of unity. It is classically known that $E$ is a principal ideal ring, so the theory of division in $E$ for ideals and for elements is the same up to the units of $E$, which are plainly the powers of $pm j$. The prime elements (ideals) of $E$ are deduced from those of $mathbf Z$ in the following manner : a prime $p$ of $mathbf Z$ does not necessarily remain prime in $E$, but it can decompose only in 3 possible ways : (1) $p$ is inert, i.e. remains prime, iff $pequiv 2$ mod $3$ ; (2) $p$ splits, i.e. $p=pibarpi$ (where $pi$ is a prime of $E$) iff $pequiv 1$ mod $3$ ; (3) $p$ ramifies in $E$, i.e. $p=pi^2$ iff $p=3$ (see any textbook in ANT).



    Now the equation $a^2-ab+b^2=c^2$ in $mathbf Z$ reads $(a+bj)(a+bj^2)=c^2$ in $E$, where the UFD property implies that the prime divisors $pi$ of $c$ are exactly such that $pi$ divides $a+bj$, say, and the conjugate $bar pi$ divides $a+bj^2$. By uniqueness again, $pibar pi$ will divide $c$, i.e. the rational primes $p$ which divide $c$ must split.



    (*) An archetypical example lies in the attempts to solve FLT by the same approach as above. Actually Kummer showed FLT along these lines - but of course with more involved arguments - in the so called regular case, i.e. when the ring $mathbf Z[zeta_p]$ (where $zeta_p$ is a primitive $p$-th root of unity) is principal.






    share|cite|improve this answer

























      up vote
      0
      down vote













      Here is an algebraic theoretical proof parallel to the "gaussian" proof of Fermat's theorem on sums of squares. You may find it too complicated, but actually the general trend in math is to try to embed particular questions into larger and larger general theories in order to solve in one stroke whole families of problems which are of the same nature (*)



      First replace the Gauss ring $mathbf Z[i]$ by the Eisenstein ring $E=mathbf Z[j]$, which is the ring of integers of the quadratic field $mathbf Q(j)$, where $j$ is a primitive 3rd root of unity. It is classically known that $E$ is a principal ideal ring, so the theory of division in $E$ for ideals and for elements is the same up to the units of $E$, which are plainly the powers of $pm j$. The prime elements (ideals) of $E$ are deduced from those of $mathbf Z$ in the following manner : a prime $p$ of $mathbf Z$ does not necessarily remain prime in $E$, but it can decompose only in 3 possible ways : (1) $p$ is inert, i.e. remains prime, iff $pequiv 2$ mod $3$ ; (2) $p$ splits, i.e. $p=pibarpi$ (where $pi$ is a prime of $E$) iff $pequiv 1$ mod $3$ ; (3) $p$ ramifies in $E$, i.e. $p=pi^2$ iff $p=3$ (see any textbook in ANT).



      Now the equation $a^2-ab+b^2=c^2$ in $mathbf Z$ reads $(a+bj)(a+bj^2)=c^2$ in $E$, where the UFD property implies that the prime divisors $pi$ of $c$ are exactly such that $pi$ divides $a+bj$, say, and the conjugate $bar pi$ divides $a+bj^2$. By uniqueness again, $pibar pi$ will divide $c$, i.e. the rational primes $p$ which divide $c$ must split.



      (*) An archetypical example lies in the attempts to solve FLT by the same approach as above. Actually Kummer showed FLT along these lines - but of course with more involved arguments - in the so called regular case, i.e. when the ring $mathbf Z[zeta_p]$ (where $zeta_p$ is a primitive $p$-th root of unity) is principal.






      share|cite|improve this answer























        up vote
        0
        down vote










        up vote
        0
        down vote









        Here is an algebraic theoretical proof parallel to the "gaussian" proof of Fermat's theorem on sums of squares. You may find it too complicated, but actually the general trend in math is to try to embed particular questions into larger and larger general theories in order to solve in one stroke whole families of problems which are of the same nature (*)



        First replace the Gauss ring $mathbf Z[i]$ by the Eisenstein ring $E=mathbf Z[j]$, which is the ring of integers of the quadratic field $mathbf Q(j)$, where $j$ is a primitive 3rd root of unity. It is classically known that $E$ is a principal ideal ring, so the theory of division in $E$ for ideals and for elements is the same up to the units of $E$, which are plainly the powers of $pm j$. The prime elements (ideals) of $E$ are deduced from those of $mathbf Z$ in the following manner : a prime $p$ of $mathbf Z$ does not necessarily remain prime in $E$, but it can decompose only in 3 possible ways : (1) $p$ is inert, i.e. remains prime, iff $pequiv 2$ mod $3$ ; (2) $p$ splits, i.e. $p=pibarpi$ (where $pi$ is a prime of $E$) iff $pequiv 1$ mod $3$ ; (3) $p$ ramifies in $E$, i.e. $p=pi^2$ iff $p=3$ (see any textbook in ANT).



        Now the equation $a^2-ab+b^2=c^2$ in $mathbf Z$ reads $(a+bj)(a+bj^2)=c^2$ in $E$, where the UFD property implies that the prime divisors $pi$ of $c$ are exactly such that $pi$ divides $a+bj$, say, and the conjugate $bar pi$ divides $a+bj^2$. By uniqueness again, $pibar pi$ will divide $c$, i.e. the rational primes $p$ which divide $c$ must split.



        (*) An archetypical example lies in the attempts to solve FLT by the same approach as above. Actually Kummer showed FLT along these lines - but of course with more involved arguments - in the so called regular case, i.e. when the ring $mathbf Z[zeta_p]$ (where $zeta_p$ is a primitive $p$-th root of unity) is principal.






        share|cite|improve this answer













        Here is an algebraic theoretical proof parallel to the "gaussian" proof of Fermat's theorem on sums of squares. You may find it too complicated, but actually the general trend in math is to try to embed particular questions into larger and larger general theories in order to solve in one stroke whole families of problems which are of the same nature (*)



        First replace the Gauss ring $mathbf Z[i]$ by the Eisenstein ring $E=mathbf Z[j]$, which is the ring of integers of the quadratic field $mathbf Q(j)$, where $j$ is a primitive 3rd root of unity. It is classically known that $E$ is a principal ideal ring, so the theory of division in $E$ for ideals and for elements is the same up to the units of $E$, which are plainly the powers of $pm j$. The prime elements (ideals) of $E$ are deduced from those of $mathbf Z$ in the following manner : a prime $p$ of $mathbf Z$ does not necessarily remain prime in $E$, but it can decompose only in 3 possible ways : (1) $p$ is inert, i.e. remains prime, iff $pequiv 2$ mod $3$ ; (2) $p$ splits, i.e. $p=pibarpi$ (where $pi$ is a prime of $E$) iff $pequiv 1$ mod $3$ ; (3) $p$ ramifies in $E$, i.e. $p=pi^2$ iff $p=3$ (see any textbook in ANT).



        Now the equation $a^2-ab+b^2=c^2$ in $mathbf Z$ reads $(a+bj)(a+bj^2)=c^2$ in $E$, where the UFD property implies that the prime divisors $pi$ of $c$ are exactly such that $pi$ divides $a+bj$, say, and the conjugate $bar pi$ divides $a+bj^2$. By uniqueness again, $pibar pi$ will divide $c$, i.e. the rational primes $p$ which divide $c$ must split.



        (*) An archetypical example lies in the attempts to solve FLT by the same approach as above. Actually Kummer showed FLT along these lines - but of course with more involved arguments - in the so called regular case, i.e. when the ring $mathbf Z[zeta_p]$ (where $zeta_p$ is a primitive $p$-th root of unity) is principal.







        share|cite|improve this answer













        share|cite|improve this answer



        share|cite|improve this answer











        answered Jul 20 at 14:18









        nguyen quang do

        7,4521621




        7,4521621






















             

            draft saved


            draft discarded


























             


            draft saved


            draft discarded














            StackExchange.ready(
            function ()
            StackExchange.openid.initPostLogin('.new-post-login', 'https%3a%2f%2fmath.stackexchange.com%2fquestions%2f2855769%2fa2%25e2%2588%2592abb2-c2-show-prime-factors-of-c-are-of-the-form-6k-1%23new-answer', 'question_page');

            );

            Post as a guest













































































            Comments

            Popular posts from this blog

            Color the edges and diagonals of a regular polygon

            Relationship between determinant of matrix and determinant of adjoint?

            What is the equation of a 3D cone with generalised tilt?